Микробиология
VI.
1. Choose medication to treat diphtheria:
   А. DPT vaccin
   B. ADT-M-anatoxins, ADT-anatoxins
   С. Antitoxin serum
   D. Monoanatoxins
   E. Diphtheria monoreceptor phages
   F. Normal Gamma globulins
2. The drug injected intradermally for determening the delayed-type hypersensitivity in case of tuberculesis is ###
   ...
3. Установите соответствие Establish a correspondence between species of microorganisms and sensitive laboratory animals: 1) Mycobacterium tuberculosis; 2) Mycobacterium bovis;3) Mycobacterium leprae;
    Rabbits
    Guinea pigs
    Armadillos
4. Choose biomaterials used for microbiological diagnostics of leprosy:
   А. Sputum, nasopharyngeal mucus, pleural exudate
   B. Blood, cerebrospinal fluid
   С. Scrapings from skin granulomas and mucous granulomas
   D. Sore discharge, transudate
   E. Conjunctiva discharge, vulva discharge
   F. Feces, urine
5. Choose main clinical forms of leprosy ###, ###
   ...
+ I. All of above are correct
6. Choose characteristics of family Enterobacteriaceae:
   А. Gram-negative rods
   B. Non-spore forming
   С. Facultative anaerobic respiration
   D. Fermenting carbohydrates to produce acid
   E. Catalase-positive
   F. Oxidase-negative
   G. A, B, C are correct
   H. A, D, F are correct
   I. All of above are correct
7. Fermentation of ### is typical for all members of Enterobacteriacea family.
   ...
8. What is the main laboratory diagnostic method for determination of acute intestinal infections caused by members of Enterobacteriaceae
   Bacteriological
9. What genus of Enterobacteriaceae causes typhoid and paratyphoid fevers, gastroenteritis and septicaemias in humans? ###
   ...
10. The natural reservoir of Salmonella Typhi bacteria is ###.
   ...
11. Specific syndrome at an the first stage of typhoid fever (increased temperature and confusions) is observed during bacteremia and caused by ### of bacteria.
   ...
12. What are possible routes of tuberculosis transmission?
   А. Droplet transmission
   B. Foodborne transmission
   С. Direct physical contact
   D. Parenteral transmission
   E. Sexual contact
   F. All of the above are correct
   G. A, B, E are correct
   H. A, B, C are correct
13. What is the differential staining method for determining Mycobacterium genus?
   Ziehl-Neelsen
14. What is the reason of Mycobacteria acid-resistance during staining procedures?
   А. Their cell wall contains a lot of lipids and wax
   B. Their cell wall contains lipopolysaccharides
   С. Their membrane contains a lot of water
   D. They contain volutin granules
15. The crucial virulence factor of Mycobacterium tuberculosis containing in the cell wall is ###, ###
   ...
16. What Mycobacterium species is unable to grow on any nutrient medium (Latin name ###, ###)?
   ...
17. Choose methods for tuberculosis diagnostics:
   А. Bacterioscopic
   B. Bacteriological
   С. Biological
   D. Allergic
   E. Genodiagnostic
   F. A, B are correct
   G. A, B, D are correct
   H. All of the above are correct
18. Choose the source of infection of typhoid fever:
   А. pets
   B. wild animals
   С. bacilli-carriers
   D. patients with typhoid fever
   E. mouse-like rodents
   F. A and B are correct
   G. C and D are correct
19. The reservoir of shigellosis causative agents is ###.
   ...
20. In human body Shigella species invade epithelial lining of ### ### causing erosions and ulcers formation.
   ...
21. Choose characteristics of dysentery causative agents:
   А. Rod-shaped
   B. Ferment carbohydrates without producing gases
   С. Do not have flagella
   D. Release hydrogen sulfide
   E. Spore forming
   F. A, B, C are correct
   G. B, C, D are correct
   H. A, B are correct
22. In human body, dysentery pathogens (Shigella) are found:
   А. Inside of colon epithelial cells
   B. On the surface of enterocytes villi
   С. In the lumen of the small intestine
23. What selective media is used for isolation of Salmonella bacteria from feces? ### ### ###
   ...
24. Choose pathogenic factors of diarrheagenic Escherichia coli:
   А. Heat-labile enterotoxin
   B. Eritrogenin
   С. Protein A
   D. Exfoliative toxin
25. Choose features of Shigella bacteria virulence:
   А. Caused by flagella motions
   B. Caused by lipopolysaccharides
   С. Caused by Shiga toxin (cytotoxin) production
   D. Caused by invasin (outer membrane protein) production
   E. Observed in the presence of calcium ions
   F. B, C, D are correct
   G. B, D, E are correct
   H. A, B, C are correct
26. Choose criteria of dividing Escherichia coli species into categories:
   А. Biochemical
   B. By set of virulence factors
   С. Cultural
   D. Morphological
   E. By sensitivity to bacteriophages
27. What is the role of non-pathogenic Escherichia coli stains normally living in the human intestine:
   А. They are antagonists of pathogenic microorganisms
   B. They determine colonization resistance
   С. They produce exotoxins
   D. They participate in proteins and lipids metabolism and bile acids transformations
   E. They participate in the certain vitamins and hormones synthesis
   F. All of above are correct
   G. A, B, D, E are correct
28. Diarrheagenic Escherichia coli stains can be divided into several categories: enterotoxigenic, enteropathogenic, enterohemorrhagic, enteroaggregative and ###.
   ...
29. Colonization factor and cholerogen-like enterotoxin are typical pathogenic factors of ### E. coli category.
   ...
30. Choose cholera causative agents:
   А. Any Vibrio cholerae
   B. Toxigenic Vibrio cholerae
   С. Any Vibrionaceae
   D. Nontoxigenic Vibrio cholerae
   E. None of above are correct
31. All morphological characteristics of Vibrio cholerae are correct except:
   А. Comma-shaped
   B. Monotrichous
   С. Non-spore forming
   D. Ovoid
   E. Capsule forming
   32. Cholera gems and choleriform vibrio are distinguished by following test except:
   А. Agglutination by О1 or О139 serums
   B. Lysis by specific bacteriophages
   С. Oxidase presence
   D. Novobiocin resistance
33. Choose biomaterials used for accelerated determining of cholera pathogens using RIF test:
   А. Pus
   B. Stool
   С. Sputum
   D. Blood serum
34. What is the role of Vibrio cholerae О139 in human pathology:
   А. Causes mild diarrhea
   B. Causes typical cholera
   С. Causes food toxic infections
   D. No role
   E. Opportunistic microorganisms
35. What is the role of specific secretory IgA in the organism of the patient with cholera:
   А. Prevent Vibrio cholerae penetration into blood
   B. Kill Vibrio cholerae in bloodstream
   С. Block Vibrio cholerae attachment to the small intestine epithelium
   D. Provide intestine purification from Vibrio cholerae
   E. Kill Vibrio cholerae in the intestinal lumen
   F. A, B are correct
   G. C, D are correct
   H. D, E are correct
36. Vibrio cholerae strains relate to the germ causing cholera by:
   А. Sugars decomposition
   B. Sensitivity to antibiotics
   С. Agglutination by О1 or О139 serums
   D. Sensitivity to diagnostic cholera monophages
   E. A, B, C are correct
   F. A, C, D are correct
   G. All of above are correct
37. Choose the feature that is not typical for Clostridium botulinum:
   А. Gram-positive
   B. Subterminal-located endospores are present
   С. Distinctive capsule is present
   D. Flagella across the surface are present
   E. Large rods
   F. Obligate anaerobic type of energetic metabolism
38. Routes of botulism transmission are foodborne and ### transmission.
   ...
I. All of above are correct
39. The major factors of botulism transmission are:
   А. Birds' eggs
   B. Preserved homemade products
   С. Fish and meat products
   D. Confectionery
   E. Soil, silt
   F. Open water
   G. A, B, D, F are correct
   H. B, C, E are correct
40. The major virulence factor of Clostridium botulinumis ###
   ...
41. Choose the biomaterial that is not used for botulism tests:
   А. Vomit and gastric washings
   B. Blood
   С. Sputum
   D. Urine
   E. Feces
   F. Leftover food
42. Blood of the patient with suspected botulism is tested to detect the presence of ###:
   ...
43. Choose medication that is used for emergency prevention of botulism:
   А. Anatoxin
   B. Antitoxic polyvalent serum
   С. Inactivated vaccine
   D. Antitoxic monovalent serum
44. For botulism serotherapy do not use:
   А. Antitoxic polyvalent serum
   B. Typical antitoxic serums A, В, Е
   С. Botulinum anatoxin
+ I. All of above are correct
45. The major factors of gastrointestinal tract dysbiosis are:
   А. Stress
   B. Unbalanced diet
   С. Intestinal infections
   D. Antibacterial drugs treatment
   E. Long hormone and chemo- and radiotherapy
   F. Immunodeficiency states
   G. A, B, C are correct
   H. D, E, F are correct
   I. All of above are correct
46. Plague pathogens are:
   А. Gram-negative cocci
   B. Ovoid bipolar-stained Gram-negative rods
   С. Gram-negative curved rods
   D. Ovoid bipolar-stained Gram-positive rods
   E. Gram-negative coccobacteria
I. A, B, D, E, F are correct
47. Virulence factors of plague bacteria are:
   А. Capsule
   B. V- and W-, F1- antigens
   С. Exotoxin (murine toxin)
   D. Nucleoproteins
   E. Endotoxin
   F. Plazmocoagulase, fibrinozin
   G. D, E, F are correct
   H. A, B, C, E, F are correct
48. Cultural properties of anthrax causative agents are:
   A. Not demanding to growth medium
   B. Grow better at 28 C
   C. Form R-form of colonies
   D. Require presence of bile in medium
   E. Grow in the atmosphere with necessary presence of carbon dioxide
   F. А, B, D are correct
   G. A, B, C are correct
   H. С, D, E are correct
49. Complex exotoxin (consisting of lethal factor, edema factor, and protective antigen) and presence of capsule are virulence factors of ### ###(Latin name).
   ...
I. A, C, E are correct
50. What are routes of anthrax transmission to humans:
   A. Sexual
   B. Aerogenic
   C. Transplacental
   D. Alimentary
   E. Contact
   G. A, C, D are correct
   H. B, D, E are correct
51. Choose the biomaterials used for laboratory diagnostics of anthrax:
   А. Animal raw material (wool, skin, meat)
   B. Sore discharge or blister fluid
   С. Sputum
   D. Feces
   E. A, B are correct
   F. B, C, D are correct
   G. All above are correct
52. What is the earlier method of tularemia diagnostics?
   Allergological
53. Choose the method of specific prevention of tularemia:
   А. Rodents elimination
   B. Risk group vaccination
   С. Bacilli-carriers elimination
   D. Bacilli-carriers and rodents elimination
   E. Universal vaccination
54. Healthy people get infected from the patients with tularemia:
   А. Seldom
   B. Almost never
   С. Often
55. Select brucellosis causative agents:
   А. Вrucella melitensis
   B. Вrucella abortus
   С. Вrucella suis
   D. Bacillus anthracis
   E. Yersinia pestis
   F. D, E are correct
   G. A, B, C are correct
   H. A, E are correct
56. Among the living organisms ### cannot be the reservoir of brucella
   ...
57. Select the impossible route of brucellosis transmission:
   А. Alimentary
   B. Contact
   С. Vector-borne
   D. Droplet
58. Brucellosis pathogens in lymphoid-macrophage system do the following:
   А. Incapsulate
   B. Reproduce
   С. Do not reproduce
59. Choose method that is not used for brucellosis diagnostics:
   А. Biological
   B. Bacteriological
   С. Serological (Wright-Huddleson test)
   D. Allergic (Burnet test)
60. What is the purpose of Wright test (agglutination reaction):
   А. Determination of brucellosis antigens in patient serum
   B. Allergic diagnostics of brucellosis
   С. Determination of brucellosis antibodies in patient serum
   D. Selection of people to vaccinate against brucellosis
   E. Determination of phagocytosis completion
61. Morphologically Leptospira bacteria are:
   А. Thin spiral microorganisms with 3-5 large uneven curvs and pointed ends
   B. Coccobacteria
   С. Thin spirals with closely adjacent curvs and end hooks. The cell has S- or C-curved shaped.
   D. Thin spirals with 8-12 even curvs
62. ### can have S- or C-curved shape due to secondary curves presence
   ...
63. Morphologically ### bacteria are thin spiral microorganisms with 3-8 large uneven curls and pointed ends.
   ...
64. The main virulence factor of Borrelia causing the relapsing fever is ###
   ...
63. Select disease caused by spirochetes:
   А. Typhoid fever
   B. Lyme disease
   С. Toxoplasmosis
   D. Q-fever
   E. Candidiasis
64. Select disease that is not caused by spirochetes:
   А. Syphilis
   B. Q fever
   С. Leptospirosis
   D. Lyme disease
   E. Relapsing fever
65. The reservoir of causative agents of epidemic relapsing fever is ###.
   ...
66. Select pathogens of Lyme disease:
   А. Borrelia recurrentis
   B. Borrelia persica
   С. Borrelia caucasica
   D. Borrelia burgdorferi
   E. Borrelia duttonii
67. The vectors of Lyme disease is ### ###
   ...
68. The main method of relapsing fever diagnostics is ###
   ...
69. What clinical biomaterials are used for relapsing fever diagnostics? ###
   ...
70. All spirochetes are Gram-###.
   ...
71. Out of all spirochetes the ### pathogens are less subjected to staining with aniline dyes.
   ...
72. Morphologically ### bacteria are thin spirals with 8-12 even curls.
   ...
73. What are reservoirs of syphilis infections? ###
   ...
74. Typical symptom of secondary syphilis is:
   А. Gumma
   B. Chancre
   С. Rash
   D. Tabes dorsalis
   E. Progressive paralysis
75. Gumma, tabes dorsalis, progressive paralysis are characteristics of ### period of syphilis.
   ...
76. Select serological reaction that is not used for syphilis diagnostics in seropositive period:
   А. Treponema pallidum immobilization test (TPI)
   B. Fluorescent treponemal antibody test (FTA)
   С. Ascoli reaction
   D. Reagin Wasserman (RW)
   E. Rapid Plasma Reagin (RPR)
   F. Enzymelynked immunosorbent assay (ELISA)
77. Select the medication for specific prevention of syphilis:
   А. Attenuated vaccine
   B. Inactivated vaccine
   С. Specific prevention is not developed
   D. Chemical vaccine
   E. Anatoxin
78. Select the medication for specific prevention of anthrax:
   А. Inactivated vaccine
   B. Chemical vaccine
   С. Live non-encapsulated spore vaccine
   D. Specific bacteriophage
   E. Specific prevention is not developed
79. Gonococci are Gram-### bacteria
   ...
80. The major route of gonorrhea transmission is ###
   ...
81. The main method of gonoblenorrhea diagnostics is ###
   ...
82. What is the major transmission route of newborns gonoblenorrhea infection?
   А. Transplacental
   B. Intrauterine
   С. Through birth canals
   D. Sexual
83. What is the main method of microbiological diagnostics of acute gonorrhea?
   А. Biological
   B. Bacterioscopic
   С. Bacteriological
   D. Serological
84. What is the main method of chronic gonorrhea diagnostics?
   А. Biological
   B. Bacteriological
   С. Serological
   D. Bacterioscopic
85. Select the reason of complete dependence of chlamidya from the host cells:
   А. Low content of nucleoproteins in the cell
   B. Inability to synthesize ATP
   С. High wax and lipid content in the cell
   D. Absence of ribosomes
86. Severe, generalized, acute or chronic febrile state with pathogens multiplication in the blood and lymphatic systems is called:
   А. Bacteremia
   B. Sepsis
   С. Toxinemia
   D. Viremia
87. Staphylococci do not cause:
   А. Purulent-inflammatory processes of the skin, lymph nodes
   B. Purulent-inflammatory processes of the respiratory system organs, eyes, sinuses
   С. Ornithosis
   D. Purulent-inflammatory processes of the central nervous system
   E. Sepsis
   F. Food poisoning
88. Staphylococci bacteria have ### ### type of respiration.
   ...
89. Staphylococci bacteria grow in liquid cultural media in the form of:
   А. Bottom sludge
   B. Diffuse clouding
   С. Wool lumps
   D. Surface film
90. Choose elective nutrient media for Staphylococcus species:
   А. Beef-extract agar, beef-extract broth
   B. Egg-yolk salt agar, beef-extract broth with 6,5% NaCl
   С. Blood agar, serum agar
   D. Endo, Levin, Ploskirev media
   E. Kitt-Tarozzi medium
91. Choose addition to egg-yolk salt agar that provides elective conditions for Staphylococcus species:
   А. Milk
   B. 6,5% or 10% NaCl
   С. Egg yolk
   D. Agar-agar
92. What Staphylococcus aureus toxin does newborn pemphigus (scalded skin syndrome) cause ? ###
   ...
93. What is the main method of Staphylococcus infections diagnostics? ###
   ...
94. A set of type-specific staphylococcal ### is used for intra-species typing of Staphylococcus aureus strains of human origin.
   ...
95. Gram-positive cocci with plasmocoagulase isolated from pus can be identified as ### ### (Latin name):
   ...
96. What biomaterials are used for bacteriological diagnostics of Staphylococcus infections:
   A. Pus
   B. Sputum, nasopharyngeal mucus
   C. Sinuses aspirates
   D. Joint fluid
   E. Blood
   F. CSF
   G. Cavities exudates
   H. All above are correct
97. Streptococcus species do not cause:
   А. Purulent-inflammatory processes of any localization
   B. Sepsis
   С. Food poisoning
   D. Meningitis
   E. Scarlet fever
   F. Toxic shock syndrome
98. Round-shaped, Gram-positive, non-spore forming, facultative anaerobic, catalase-negative, non-motile, chains in smears are characteristics of ### bacteria
   ...
99. What type of hemolysis on blood agar is typical for most of Streptococcus pyogenes strains? ###
   ...
100. What type of hemolysis on blood agar is typical for most of Streptococcus pneumonia strains? ###
   ...
101. ### is virulence factor of Streptococcus pyogenes causing death of polymorphic nuclear leukocytes and macrophages.
   ...
102. ### is virulence factor of Streptococcus pyogenes that is formed in vivo and protects it from phagocytosis.
   ...
103. Streptococcus pyogenes caused Scarlet fever differs from other Streptococci of serogroup A in ### toxin producing
   ...
104. The major pathogen of gas anaerobic infection is:
   А. Clostridium septicum
   B. Clostridium perfringens
   С. Clostridium novyi
   D. Clostridium tetani
105. Clostridium perfringens bacteria differ from other gas clostridia pathogens in ### absence
   ...
106. Clostridium perfringens bacteria in addition to essential structures have:
   А. Volutine inclusions
   B. Capsule
   С. Terminal located spores
   D. Flagella
   E. Subterminal and central located spores
   F. a, d are correct
   G. b, e are correct
   H. a, c, are correct
107. ### ### is used for emergency prevention and specific therapy of gas gangrene.
   ...
108. The major source of infection of Clostridium tetani is:
   А. Air
   B. Soil
   С. Water
   D. Human body
   E. Animals and human blood vessels endothelium
109. Tetanus causative agents have ### type of respiration.
   ...
110. Choose medications that are used for specific prevention of tetanus:
   A. BCG vaccine
   B. Fermi vaccine
   C. DTP vaccine
   D. Antitoxin serum
   E. DT-anatoxins
   F. All of above are correct
   G. C, D, E are correct
   H. A, B, C are correct
111. Choose medication that is used for passive emergency prevention of tetanus:
   А. DTP or DT - vaccines
   B. Antitoxin serum or gamma-globulin
   С. Bacteriophages
   D. Tetanus anatoxin
112. Установите соответствие Establish a correspondence between species of microorganisms and cell form and spore location: 1). Clostridium perfringens 2). Clostridium botulinum 3). Clostridium tetani
    Central located spore, fusiform cell
    Terminal located spore, “drum stick” cell
    Subterminal located spore, “tennis racket” cell
113. Choose possible reservoirs of meningococcal infection :
   А. Patients with meningococcal infections
   B. Bacteria-carrier humans
   С. Pets with meningococcal infections
   D. Wild animals with meningococcal infections
   E. Bacilli-carrier wild animals
   F. A, B, are correct
   G. A, D are correct
   H. D,E are correct
114. What is the major reservoir of meningococcal infections? ###-###
   ...
115. Choose characteristics of Meningococci:
   А. Gramm-negative
   B. Bean-shaped
   С. Pairwise arrangement in the smear
   D. Oxidase- and catalase-positive
   E. Fermenting glucose and maltose to produce acid
   F. A, B, C are correct
   G. C, D, E are correct
   H. All of above are correct
116. Meningococci have ### type of respiration.
   ...
117. Bordetella pertussis bacteria have ### type of respiration.
   ...
I. All above are correct
118. Choose microbiological methods of meningococcal disease diagnostics:
   А. Bacterioscopic
   B. Bacteriological
   С. Serological
   D. Biological
   E. Phagotyping
   F. Allergological
   G. D, E,F are correct
   H. A, B, C are correct
119. What biomaterial is used for laboratory microbiological diagnostics of meningococcemia? ###
   ...
120. What biomaterial is used for laboratory determining of meningococcal bacteria-carriage? ###
   ...
121. What pathological biomaterial is used for laboratory meningococcal meningitis diagnostics? ### ###
   ...
123. Установите соответствие Establish a correspondence between meningococcal disease and pathological biomaterial taken for laboratory diagnosis:1) Meningococcemia 2) Nasopharyngitis, meningococcal-carriage 3) Meningitis
    Blood
    CSF (cerebro-spinal fluid)
    Nasopharyngeal mucus
I. D, F are correct
124. The Pertussis causative agents are:
   А. Lancet-shaped cocci
   B. Large rods
   С. Coccobacteria
   D. Gram-negative microorganisms
   E. Streptobacillas
   F. Gramm-positive microorganisms
   G. C, D are correct
   H. A, F are correct
125. Select biotop of whooping cough causative agents:
   А. Upper respiratory tract of domestic and wild animals
   B. Upper respiratory tract of patients with whooping cough
   С. Mucous membrane of the human intestine
   D. Mucous membrane of the intestines of animals
   E. Human skin
126. What is the transmission route of whooping cough? ###
   ...
127. Select biochemical activity of Bordetella pertussis:
   А. Biochemically inactive
   B. Ferment glucose and mannit
   С. Ferment maltose and saccharose
   D. Urease-positive
   E. Tirosin-positive
   F. Citrate utilizer
128. Microcapsule, fimbriae, pertactin, endotoxin and tracheal cytotoxin, adenylate cyclase and filamentous hemagglutinin are virulence factors of ### ### (Latin name) bacteria.
   ...
I. C, E are correct
129. Choose the major methods of pertussis diagnistics:
   А. Bacteriological
   B. Biological
   С. Serological
   D. Microscopic
   E. Allergical
   F. All of above are correct
   G. A, C are correct
   H. B, D are correct
130. ### method is used for pertussis diagnostics during the 1-st week.
   ...
131. What pathological biomaterial is used for bacteriological method of pertussis diagnostics?
   А. Blood, CSF
   B. Respiratory tract secretion, sputum
   С. Nose mucous membrane scrapings
   D. Necrotic tissue
132. Choose the medication that is used for specific prevention of pertussis:
   А. BCG vaccine
   B. DPT-vaccine
   С. ADT-anatoxin
   D. Sexta-anatoxin
   E. Specific Gamma globulins
133. Choose diphtheria pathogens biovars:
   А. Gravis
   B. Mitis
   С. Xerosis
   D. Pneumoniae
   E. A, B are correct
   F. C,D are correct
   G. All of above are correct
134. Choose morphological characteristics of diphtheria causative agents:
   А. Gram-negative coccobacteria, capsule, non-spore forming, flagellum absence
   B. Gram-positive, medium sized clavate rods, non-spore forming, flagellum absence
   С. Gram-positive, fusiform large rods, spore- and capsule-forming, flagellum absence
   D. Gram-positive large rods, no capsule-forming, terminal-located spores, peritrichous
135. What is the major route of diphteria transmission?
   Droplet
136. In smears Corynebacterium diphtheriae stains are situated:
   А. Chaotic
   B. Placed at an angle to each other (X- and V-shaped)
   С. Pairwise
   D. Chain-like
   E. In the form of “Cigarette packs”
   F. In the form of “Paling”
137. Diphteria pathogens are club-shaped due to presence of ### granules at the cell poles.
   ...
138. Corynebacterium diphtheriae gravis on tellurite blood agar grow as:
   А. S-form colonies
   B. Small smooth convex colonies black in color, with smooth edges
   С. Small smooth convex colonies, grayish in color, mercury-drop-like
   D. R-form colonies, grayish-black in color with radial striations and uneven edges
   E. Daisy-flower-shaped
139. Corynebacterium diphtheriae mitis on tellurite blood agar grow in the form of
   А. “Pebbled leather”
   B. Small smooth convex colonies black in color, with smooth edges
   С. Crumbling colonies, grayish-black in color with radial striations and uneven edges
   D. Daisy-flower-shaped
   E. Small smooth convex colonies, grayish in color, mercury-drop-like
140. Diphteria toxin causes:
   А. Pulmonary edema, severe hypoxia, apnea
   B. Direct lesions of the nervous tissue and spasmodic contraction of striated muscle
   С. Adrenal glands, myocardium and periferical nervous system lesions
   D. Vision loss, afonia, apnea due to inhibition of acetylcholine releasing in the synapses
141. What is the major method of microbiological diagnostics of diphtheria? ###
   ...
142. Choose localization of the pathological processes which do not caused by Corynebacterium diphtheria bacteria:
   А. Skin, wounds
   B. Eye conjunctiva, ears
   С. Pharynx, tonsils, nose
   D. Vagina
143. Choose the differential diagnostic test in bacteriological diagnostics of the disease “diphtheria”:
   A. Toxinogenicity test
   B. Tellurite blood agar growth test
   С. Zaks test
   D. Pizu test
   E. Sugars decomposition test
144. ### reaction is used for enterobacteria serotyping.
   ...
145. Lactose fermentation is typical for:
   А. Escherichia coli;
   B. Shigella flexneri;
   С. Salmonella Typhi;
   D. Salmonella Typhimurium.
145. O-antigenes of enterobacteria are ### according their chemical structure.
   ...
146. H-antigenes of enterobacteria are ### according their chemical structure.
   ...
147. What enterobacteria genus includes resident bacteria of human body microbiota?
   Escherichia
148. Enterobacteria species are Gram-###.
   ...
149. Enterobacteria have ###-shaped cells.
   ...
150. Choose enterobacteria respiration type:
   А. Aerobic
   B. Anaerobic
   С. Facultative anaerobic
   D. Microaerophils
151. All enterobacteria species can utilize:
   А. Glucose
   B. Lactose
   С. Sucrose
   D. Mannite
152. Pathogenic E. coli strains differ from opportunistic E. coli strains in:
   А. Color of colonies on Endo agar
   B. Antigenic properties
   С. Lactose fermenting ability
   D. Glucose fermenting ability
153. Clinics and pathogenesis of diseases caused by enteroinvasive E. coli are similar to clinics and pathogenesis of ###.
   ...
154. Clinics and pathogenesis of diseases caused by enterotoxigenic E. coli are similar to ### clinics and pathogenesis.
   ...
155. What antigen does define the serogroups of E.coli? ### (letter)
   ...
156. What is the major method of shigelosis diagnostics? ###
   ...
157. The causative agents of bacillary dysentery belong to ### genus.
   ...
158. What is binomial Latin name of Gram-negative bean-shaped diplococcus causing venereal diseases? ### ###
   ...
159. What is binomial Latin name of food poisoning pathogens resembling tennis racket, with subterminal-located spores? ### ###
   ...
160. What is binomial Latin name of trachoma (person-to-person conjuctivitis) pathogen forming elementary and reticular bodies? ### ###
   ...
161. What is binomial Latin name of causative agents of ricketsiosis transmitted by lice? ### ###
   ...
162. What is binomial Latin name of Gram-negative diplococci that cause meningitis? ### ###
   ...
163. What is binomial Latin name of typhoid fever germ that do not produce gas when hemoculture is isolated on liquid (Rappaport) medium? ### ###
   ...
164. What is binomial Latin name of bubonic type of infection pathogen that have “safety pin”-shape and spread by rodents? ### ###
   ...
165. What is binomial Latin name of causative agents of disease characterized by sudden dehydration and rice-water stool? ### ###
   ...
166. What is binomial Latin name of Mycobacterium species causing generalized primary chronic disease discovered by G.А.Hansen? It does not grow on cultural media.### ###
   ...
167. What is binomial Latin name of spore- and capsula-forming streptobacillus that causes disease characterized by carbuncles with brown-black crust? ### ###
   ...
168. What is binomial Latin name of pathogen that has gram-positive club-shaped form and can be finally determined by positive cystinase test? ### ###
   ...
169. What is binomial Latin name of causative agents of disease diagnosed by tularinum allergic reaction? ### ###
   ...
170. What is binomial Latin name of pathogens that can be determined by Wright-Huddleson serological reaction? Pathogens are small gram-negative coccobacilli transmitted by sheep and goats and never by human. ### ###
   ...
171. What is binomial Latin name of causative agents of visceral leishmaniasis? ### ###
   ...
172. What is binomial Latin name of causative agents of amoebiasis? ### ###
   ...
173. What is binomial Latin name of causative agents of toxoplasmosis?
   Toxoplasma gondii
1. What is the Latin name of HIV genus? ###
   ...
2. Influenza virus ultra-structure includes:
   А. Fragmented RNA
   B. Nonfragmented RNA
   С. Double-stranded RNA
   D. DNA
   E. Capsid
   F. Supercapsid
   G. B, C, E are correct
   H. A, E, F are correct
3. Choose the major antigens of influenza virus types A and B:
   А. Hexon-antigen
   B. Hemagglutinin (HA)
   С. Neuraminidase (NA)
   D. Ribonucleoprotein (RNP)
   E. M-antigen (matrix protein associated with the NP)
   F. Fusion and hemolysis proteins
   G. B, C, D, E are correct
   H. A, F are correct
4. Hemagglutinin and neuraminidase of influenza virus are ### located antigens.
   ...
5. Ribonucleoprotein (RNP) and M-matrix protein of influenza virus are ### located antigens.
   ...
I. B, D, E,G are correct
6. Choose properties of hemagglutinin (H-antigen) of influenza virus:
   А. Sticks red blood cells together, is detected by RA and RIHA
   B. Is detected by its reaction with sialic acids
   С. Induces protective immunity in organism
   D. Conservative
   E. Induces non-protective antibodies formation
   F. Undergoes antigenic shift and antigenic drift
   G. Participates in the process of virus adsorption onto the cell
   H. A, C, F, G are correct
   7. The types of influenza virus are ###, ###, ### (letters)
   А
8. Influenza A virus caused human diseases is divided into subtypes, except:
   А. А (H1N1)
   B. А (H3N3)
   C. А (H2N2)
   D. А (H3N2)
9. What test is used to determine types of influenza virus? ### ###
   ...
10. Choose features of immune response to influenza after vaccination:
   А. Does not form
   B. Type-specific
   С. Is formed in presence of antibodies to hemagglutinin and neuraminidase
   D. Is formed in presence of antibodies to ribonucleoproteins
   E. Depends significantly on the presence of secretory immunoglobulins A
   F. Intense, long lasting
   G. B, C, E are correct
   H. A, D, F are correct
11. Pandemics, epidemics are caused by influenza virus type ###. (letter)
   ...
12. Local outbreaks are caused by influenza virus type ###.(letter)
   ...
13. Sporadic diseases are caused by influenza virus type ###.(letter)
   ...
14. Laboratory diagnostics of influenza virus do not include:
   А. Viroscopy, RIA, ELISA
   B. Virological method (virus isolation)
   С. Allergic method
   D. Serological method
15. Biomaterials that used for viroscopical and virological diagnostics of influenza ### swabs.
   ...
16. Choose biomaterials that are used for serological diagnostics of influenza?
   А. Two blood samples (double serum)
   B. A blood sample (serum)
   С. Nasopharyngeal washings
   D. Feces
17. Choose medication that is used for active specific prevention of influenza:
   А. Live intranasal vaccine
   B. Inactivated virion vaccine
   С. Subunit vaccine
   D. Sabin vaccine
   E. Salk vaccine
   F. A, B, C are correct
   G. B, C, D are correct
   H. C, D, E are correct
18. Measles virus contains a negative-sense spiral single-stranded ### (abbreviation of nucleic acid type).
   ...
19. Antigens of measles virus show
   A. high variability
   B. No variability
20. What is not the characteristic of measles virus:
   А. A, B, C serotypes
   B. Serotypes are not detected
   С. Antigens unity of viruses from different geographical areas is observed
21. Measles virus can cause cytopathlogical effect during cultivation in cellular cultures such as ### formation.
   ...
22. Such complications as multiple sclerosis and subacute sclerosing panencephalitis (SSPE) can develop several years after ### infection.
   ...
23. Choose medication that is used for active prevention of measles in Russia:
   А. Inactivated vaccine
   B. Hemagglutinin-split vaccine
   С. Live vaccine
24. Polioviruses , Coxsackievirus and ECHO virus belong to the genus ### in the family Picornaviridae.
   ...
+ I. A, C, G are correct
25. Choose morphological and chemical characteristics of polioviruses:
   А. Contain positive-sense RNA genome
   B. Contain negative-sense RNA genome
   С. Have capsid with icosahedral symmetry
   D. Have outer membrane
   E. Viral particle size is 300-400 nm
   F. Bullet shape
   G. Spherical shape
   H. B, D, F are correct
   I. A, C, G are correct
26. List the types of poliovirus: ###, ###, ###
   ...
27. Choose the impossible route of poliomyelitis transmission:
   А. Fecal-oral
   B. Vector-borne
   С. Droplet
   D. Food-borne (water, milk and butter consumption)
28. Poliovirus is most likely to be isolated from ###
   ...
29. Select characteristics of immunity in case of poliomyelitis:
   А. Type-specific
   B. Humoral
   С. Cell-mediated
   D. Forms with significant participation of secretory immunoglobulins A
   E. Forms with DTH T-effectors significant participation
   F. A, C, E are correct
   G. A, B, C are correct
30. Live vaccine for poliomyelitis prevention is given ###.
   ...
31. Choose vaccine for active specific prevention of enterovirus infections that is not invited:
   А. Inactivated poliovirus vaccine
   B. ECHO virus vaccine
   С. Live poliovirus vaccine
32. Choose virus that is not causative agent of viral hepatitis:
   А. Hepatitis virus exept hepatitis A and B
   B. Hepatitis A virus
   С. Epstein-Barr virus
   D. Hepatitis B virus
   E. Hepatitis D virus
   F. Hepatitis C virus
   G. Hepatitis E virus
   33. What is the reservoir of hepatitis A virus? ###
   human
34. Choose methods that are used for hepatitis A diagnostics:
   А. Viroscopy (IEM)
   B. Virusological method, cell culture contamination
   С. Serological method, specific IgM determination
   D. HBs antigens determination
   E. Virus determination by ELIZA, RIA
   F. A, C, E are correct
   G. B, C, D are correct
35. What type of hepatitis virus belongs to the family of Hepadnaviruses, contains DNA, is transmitted parenterally, is not cultivated in cell culture, is oncogenic: ### (letter)
   ...
36. HBs antigen of hepatitis B virus is ### located antigen.
   ...
37. All characteristics of hepatitis B virus are correct, exept:
   А. Is not inactivated by treatment at 60o C for several hours
   B. Is inactivated by treatment at 60o C for several hours
   С. Is not inactivated by treatment at 100o C for 15-20 minutes
   D. UV resistant
   E. Formalin sensitive
   F. Detergents sensitive
38. Select characteristics of immuty in case of hepatitis B:
   А. Humoral
   B. Cell-mediated
   С. The role of antibodies to HBsAg is significant
   D. Antibodies to HBsAg are not protective
   E. Protects from repeated infection
   F. Does not protect from repeated infection
   G. B, D, F are correct
   H. A, C, E are correct
39. Hepatitis D virus superinfection occurs in the presence of hepatitis ### (letter) virus.
   ...
40. The rabies virus belongs to Lyssavirus genus in the ### family.
   ...
41. A negative-sense single-stranded unfragmented RNA, nucleocapsid with spiral symmetry, outer-shell, capsid proteins and matrix proteins, RNA-polymerase, glycoprotein and a bullet shape are characteristics of ### virus.
   ...
42. What is not the reservoir of Rabies virus in nature:
   А. Rodents
   B. Infected dogs, foxes, wolves
   С. Bats, jackals, cats, bobcats
43. Rabies virus can be transmitted throught ### of infected animal.
   ...
44. Rabies incubation period when the virus penetrates through head and face skin is:
   А. 10-14 days
   B. More than 1,5 month
45. Special cylinders virus promotion from sensitive peripheral nerves endings to perineural spaces and brain and spinal cord neurons and virus reproduction in neurons with formation of Babesh-Negrie cytoplasmic inclusions play significant role in pathogenesis of ### infection.
   ...
46. “Fixed” rabies virus invented by Pasteur for vaccination differs from wild rabies virus in (all are correct exclude):
   А. Plaque formation in the chorion-allantoic shell of infected chicken embryos
   B. Reproduction only in brains of rabbits
   С. Non-pathogenicity for people and dogs
   D. Causing infection with stable incubation period when passaging in rabbits brains
47. Choose indications for antirabic immunoglobulin treatment:
   А. Multiple bites, dangerously-located (head, neck)
   B. Anamnesis data about past rabies infections
48. HIV genome includes two single-strand ### molecules. (abbreviation)
   ...
49. HIV virion membrane contains glycoproteins gp with molecular masses of ### and ###.
   ...
50. Gp 120 of HIV virion attaches to the human cell membrane in the presence of ### receptor on its surfase.
   ...
51. HIV penetration to the cell cannot be achieved by:
   А. Virogeny
   B. Membranes fusion
   С. Receptor endocytosis
52. HIV infection specifically affects ### system of the organism.
   ...
53. What are impossible routes of HIV transmission? ###, ###
   ...
54. Choose cells that are not HIV target-cells:
   А. T-helper cells
   B. Monocytes, macrophages
   С. Hepatocytes
   D. Langerhans cells
   E. Astrocytes, endothelial cells
55. Choose biomaterials that contains insufficient amount of HIV:
   А. Blood
   B. Semen
   С. Vaginal and cervical discharge
   D. Brest milk
   E. Saliva, urine, lacrimal fluid
56. The goal of HIV infection therapy is:
   А. Prevention of disease progression
   B. Complete curing
   С. The elimination of the virus from the body
57. Name the form of a virus that exists outside the host cell: ###
   ...
58. Phased papular rash is the characteristic of ### virus infection.
   ...
59. ### type of virus and eukaryotic cell interaction can be characterized by viral DNA integration into the chromosome of the cell.
   ...
60. What Ag is the major marker of hepatitis B infection?###
   ...
61. What causative agents of infectious diseases do not have cellular structure?
   Virus
62. What Family of viruses contain virions with reverse transcriptase? ###
   ...
63. HIV is tropic for ### subpopulations of T-lymphocytes.
   ...
64. What laboratory test is the most reliable test in HIV infection determination, an "expert" technique? ###
   ...
65. Choose the reason of liver cells cytolysis in case of viral hepatitis B:
   А. The impact of the virus on hepatocytes
   B. Immune response to viral antigens and autoantigens
   С. Bile ducts lesions
66. Chronization of pathological process causing cirrhosis is characteristic of viral hepatitis ###.
   ...
67. Choose characteristic of antigenic drift of influenza pathogens:
   А. Antigenic variations of the viral hemagglutinins are insignificant
   B. Is associated with insignificant antigenic variations of viral capsid proteins
   С. The main reason of pandemics
   D. The result of recombination of human and bird influenza viruses
   E. Causes new antigenic types of influenza viruses formations
69. The productive interaction of the virus with the cell results in ### for the cell.
   ...
70. Influenza viruses are mostly cultivated on ### ###.
   ...
71. ### test is used to determine influenza virus in infected chicken embryos.
   ...
72. What infection disease may be complicated with orchitis in boys?
   Mumps
73. Damage of medulla oblongata motor neurons and the front horns of spinal cord is typical for ### pathogenesis.
   ...
74. Poliomyelitis vaccine injection provides prevention of ### forms of infection.
   ...
75. What type of nucleiс acid is typical for hepatitis B virus? ### (abbreviation)
   ...
76. Hepatitis C virus belongs to ### family.
   ...
77. What type of nucleiс acid presents in hepatitis C virus? ### (abbreviation)
   ...
78. Hepatitis C virus can hide from immune system due to its high ###.
   ...
79. Multiyear latent progress is characteristic of hepatitis ### virus infection. (letter)
   ...
80. What type of hepatitis virus requires assistance of helper-virus? ### (letter)
   ...
81. What is the reservoir of parental virus hepatitis? ###
   ...
82. Gp120 and gp41 are antigens of ### virus. (abbreviation)
   ...
83. What HIV antigen provides interaction with target-cells? ###
   ...
84. Indication of HIV infection is required ### determination.
   ...
85. What type of nucleiс acid presents in influenza virus? ### (abbreviation)
   ...
86. Hepatitis B virus testing of donated blood is carrying out through ### antigen determination.
   ...
87. What is the major route of hepatitis A virus transmission?###-###
   ...
88. Bacteriophages that lyse only one group of bacteria within the species are called ### bacteriophages
   ...
89. Choose ferments of HIV pol-complex:
   А. Protease, integrase, RNA-ase
   B. DNA-polymerase, integrase
   С. DNA-polymerase, protease
   D. Reverse transcriptase, RNA-ase, DNA-polymerase, integrase
90. Choose the measles symptoms:
   А. Jaundice, liver and spleen increase
   B. Phased skin rash
   С. Parotid salivary glands swelling
   D. Paralysis development
91. What is not typical for hepatitis D virus:
   А. Defective RNA containing virus
   B. Reproduction is possible only during presence of hepatitis B virus
   С. The virus monoinfection is possible
   D. The disease develops as coinfection or superinfection in presence of hepatitis B virus
92. Choose the virus that can be isolated from the feces 3 weeks after infection:
   А. Hepatitis B virus
   B. Poliovirus
   С. Influenza virus
   D. Measles virus
93. What ferment presents in the core of the influenza virus?
   А. Reverse transcriptase
   B. DNA-polymerase
   С. RNA- polymerase
   D. Protease
94. Hydro- and aerophobia, hallucinations, alternation of excitement and depression are the features of ###
   ...
1. Diploid number of chromosomes, nuclear membrane, mitochondria, Golgi apparatus, nucleolus and histone proteins are characteristics of ### microorganisms.
   ...
2. Choose main groups of microorganisms that belong to prokaryotes:
   A. Toxoplasma
   B. Actinomycetes
   C. Rickettsia
   D. Candida
   E. Chlamydia
   F. Spirochetes
   G. B, C, E, F are correct
   H. A, C, D, E are correct
   I. A, B, C, F are correct
3. Definition of morphological, tinctorial, enzymatic, antigenic, cultural, and bacteriophage sensitivity properties is used for ### of microorganisms’ species.
   ...
4. Choose method that is not used for infectious disease diagnostics in medical microbiology:
   A. Microscopic
   B. Biotechnological
   C. Biological
   D. Allergic
   E. Serological
   F. Bacteriological
5. Fixation of microorganisms to the glass during microscope slide preparation is used for:
   A. Bacteria inactivation (killing)
   B. Tight attachment of gems to the glass
   C. Better perception of dye
   D. all wrote above
6. A single chromosome, peptidoglycan, a binary type of division and absence of nuclear membrane are characteristics of ###.
   ...
7, What essential component of bacterial cell is responsible for storing genetic information and cell division?
   Nucleoid
1. Nucleoid, cytoplasm, cytoplasmic membrane, mesosomas, ribosomes are essential structures of ### cell:
   ...
2. Choose essential components providing vital activity of bacterial cell:
   A. Ribosomes
   B. Capsule
   C. Flagella
   D. Pili
   E. Spores
   F. Inclusions
3. Choose non-motile microorganisms:
   A. Vibrio
   B. Spirochetes
   C. Streptococci
   D. Salmonella
   E. Most of Clostridia
   F. Most of Escherichia
4. Bacterial ### are built from protein called flagellin.
   ...
5. According to location of flagella the fastest group of flagellated microorganisms is ###.
   ...
6. According to location of flagella the slowest group of flagellated microorganisms is ###.
   ...
7. Choose basic forms of microorganisms:
   A. Spherical
   B. Rod-shaped
   C. Spiral
   D. Star-shaped
   E. All above are correct
   F. A, B, D are correct
   H. A, B, C are correct
   G. A, B are correct
8. Choose bacteria that are not spherical (cocci):
   A. Staphylococcus aureus
   B. Brucella abortus
   C. Streptococcus pyogenes
   D. Neisseria meningitidis
9. Meningicocci of ### genus are diplococci.
   ...
10. In microscopic slides Streptococci are arranged in ###.
   ...
11. In microscope slides Staphylococci are arranged:
   A. Randomly
   B. In pairs
   С. In grape-like clusters
   D. In tetrads
   E. In the form of packages
   F. In chains
I. A, D are correct
12. Choose rod-shaped microorganisms:
   A. Enterobacteria
   B. Spirochetes
   C. Bacillus, Clostridium
   D. Neisseria
   E. Actinomycetes
   F. Yeast-like microorganisms
   G. G. A, C are correct
   H. C, D, E are correct
13. In microscope slides ### causative agents are arranged in bamboo-sticks.
   ...
14. Treponema, Borrelia and Leptospira have ### morphological shape.
   ...
15. What structure of the bacterial cell have following functions: permeability, osmotic barrier, transport, mesosoma formation, participation in metabolism and sporulation? ### ###
   ...
16. The maintenance of cell shape of bacteria is provided by ### ### of the cell.
   ...
17. Choose noncorrect characteristics of L-form of microorganisms:
   A. Partial or complete absence of the cell wall
   B. Sensitivity to bacteriophages
   C. Spherical form
   D. Ability to reproduce
   E. The possibility to recover the ability to synthesize cell wall
18. The difference of bacteria in Gram staining is due to chemical composition and structure of ### ###.
   ...
19. What substance (bioheteropolymer) is the basis of bacteria cell walls?
   Peptidoglycan
20. Choose chemical components of cellular wall of Gram-negative microorganisms:
   A. Multilayer peptidoglycan
   B. Teichoic acids
   C. Thin peptidoglycan
   D. Lipopolysaccharide
   E. Histone proteins
   F. Dipicolinate acids
   G. A, D, E are correct
   H. B, D, F are correct
   I. C, D are correct
I. A, D, E are correct
21. Choose chemical components of cellular wall of Gram-positive microorganisms:
   A. Monolayer peptidoglycan
   B. Teichoic acids
   C. Thick peptidoglycan
   D. Dipicolinate acids
   E. Lipopolysaccharide
   F. Histone proteins
   G. G. B, C are correct
   H. A, C are correct
22. The mechanism of Gram-negative bacteria staining includes:
   A. Formation of the complex of gentian violet and Lugol's solution, which cannot be washed out of multilayer peptidoglycan by alcohol
   B. Oxidation of the surface structures by Lugol's iodine
   C. Formation of the complex of gentian violet and Lugol's solution, which can be washed out of a layer peptidoglycan easily. The cell can be stained with fuchsine
   D. Denaturation of protein components with ethyl alcohol, after which the aniline dyes are not perceived
   E. Destruction of the outer membrane with 5% sulfuric acid
   F. Formation of strong complex of fuchsine and peptidoglycan which cannot be washed out by alcohol
23. The mechanism of Gram-positive bacteria staining includes:
   A. Formation of the complex of gentian violet and Lugol's solution, which cannot be washed out of multilayer peptidoglycan by alcohol
   B. Oxidation of the surface structures by Lugol's iodine
   C. Formation of the complex of gentian violet and Lugol's solution, which can be washed out of multilayer peptidoglycan easily. The cell can be stained with fuchsine
   D. Denaturation of protein components with ethyl alcohol, after which the aniline dyes are not perceived
   E. Destruction of the outer membrane with 5% sulfuric acid
   F. Formation of strong complex of fuchsine and peptidoglycan which can not be washed out by alcohol
24. Choose substance that is not used in Gram staining method:
   A. Methylene blue
   B.Fuchsine
   C.Gentian violet
   D.Lugol's iodine
   E.Ethyl alcohol
I.B, F are correct
25. Choose substances forming complex that cannot be washed out of peptidoglycan during staining of Gram-positive microorganisms:
   A.Methylene blue
   B.Lugol's iodine
   C.Pfeiffer fuchsine
   D.Ziel fuchsine
   E.Ethyl alcohol
   F.Gentian violet
   G. B, F are correct
   H.A, C are correct
26. Choose Gram-negative cocci:
   A.Staphylococci
   B.Streptococci
   C.Sarcina
   D. Meningococci and gonococci
   E.Enterococci
27. Choose Gram-positive microorganisms:
   A.Escherichia
   B.Salmonella
   C. Mycobacteria
   D.Bordetella
   E.Brucella
28. Choose Gram-negative microorganisms:
   A. Yersinia
   B.Bacillus anthracis
   C.Corynebacteria
   D.Clostridia
   E.Mycobacteria
29. Gram-positive rods placed at an angle to each other forming letters X and Y is characteristic of ### infection causative agents.
   ...
30. Установите соответствие Establish a correspondence between microorganism species, shape and Gram staining:1) Treponema pallidum, 2) Vibrio cholerae, 3) Mycobacterium tuberculosis
    Gram-positive rods
    Gram-negative spirals
    Gram-negative curved rods
31. Choose acid-fast microorganisms:
   A.Staphylococcus aureus
   B.Clostridium perfringens
   C.Mycobacterium leprae
   D.Escherchia coli
   E.Mycobacterium tuberculosis
   F.Bacillus anthracis
   H. C, E are correct
   G.B, E are correct
   I.All of above are correct
32. What property of microbial cells is due to high content of lipids, wax, phtionic acids, mycolinic, mycolic acids and tuberculostearinic acids in the cell wall? ### ###
   ...
I. C, E are correct
33. Choose substances used during Ziel-Neelson staining:
   A. Gentian violet
   B. Methylene blue
   C. Carbolic fuchsine
   D. Ethyl alcohol
   E. 0.5 % acetic acid
   F. 5 % sulfuric acid
   G. B, C, F are correct
   H. A, C, D are correct
34. During Ziel-Neelson staining, carbolic fuchsine and heating are used to:
   A.Kill microorganisms
   B.Attach microorganisms to the glass
   C. Loosen a tight cell wall and facilitate the penetration of dyes and their interaction with a cell
   D.Neutralize previously applied dye
   E.Make a cell wall more sensitive to the action of inorganic acids
35. What component of the bacterial cell, when stained by Burri-Gins, appears as colorless bezel around the red bodies of the bacteria in black background? ###
   ...
36. The functions of bacterial ### are protection from drying, phagocytosis and antibodies action.
   ...
37. Choose possible materials of bacterial capsule:
   A. Polysaccharides
   B. Lipid
   C. Peptide
   D. Lipopolisaccharide
   E. Histone proteins
   F. A, C are correct
   G. B, D are correct
   H. All above are correct
38. Capsules of most of the microorganisms consist of ###.
   ...
39. Microcapsule differs from macrocapsule in:
   A. Thickness
   B. Detection by Burri-Gins method
   C. Lipopolysaccharides
   D. A, B are correct
   E. All above are correct
40. Choose characteristics of macrocapsule (true capsule):
   A.Size is larger than the diameter of the bacterial cells
   B.Ordered fibrillar structure
   C.Tight adhesion to the cell wall
   D.Identification by Burri-Gins stain
   E.Nucleoproteids composition
   F. A, B, C, D are correct
   H.All of above are correct
41. Choose characteristics of Burry-Gins staining:
   A. Ink and fuchsine are applied on fixed microscope slide
   B. A drop of culture is mixed with the ink. Fixed smear of the mixture is stained with fuchsine
   C. Fixed microscope slide is treated with acetic methylene blue. Then slide is stained with chrysoidin
   D. A “hanging” or “crushed” drop with the addition of fuchsine is prepared
I.C, D are correct
42. Choose substances used during Burry-Gins staining:
   A.Gentian violet
   B.Methylene blue
   C.Ink
   D.Chrysoidin
   E.Diluted fichsine
   F.Ethyl alcohol
   H. C, E are correct
   G.A, F are correct
43. ### in bacterial cells are components which are responsible for energy and nutrients storing.
   ...
44. Diphtheria rods contain volutine grains:
   A. On the poles of the cell in the cytoplasm
   B.In the center of the cell in the cytoplasm
   C.In the cell wall
   D.In nucleoide
45. Choose microorganisms which are not obligate intracellular parasites:
   A.Rickettsia
   B. Mycobacteria
   C. Chlamydia
   D.Viruses
   E.Toxoplasma
46. Chlamydia and Rickettsia are obligating intracellular parasites because:
   A. They grow only on media with human blood or serum
   B. They are not able to synthesize high-energy compounds so energy metabolism is carried out only inside host cells
   C. They do not have their own metabolic activity
   D. They are not able to reproduce by binary fission
47. Choose structural components that Rickettsia do not have:
   A. Nucleoid
   B. Ribosomes
   C. Flagella
   D. Cytoplasm
   E. Cell wall
   F. Cytoplasmic membrane
48. Choose disease that is caused by Rickettsia:
   A. Typhoid fever
   B. Epidemic typhus
   C. Relapsing fever
   D. Whooping cough
   E. Diphtheria
   F. Trachoma
I. All of above are correct
49. Choose characteristics of Chlamydia infection causative agents:
   A. Nucleoid
   B. Cytoplasmic membrane, cytoplasm
   C. Ribosomes
   D. Distinct capsule
   E. Flagella, pili
   F. Elementary and reticular bodies
   G. A, B, D, E are correct
   H. A, B, C, F are correct
50. Choose functions of bacterial spores:
   A. Way of reproduction
   B. Nutrients storing
   C. Preservation of species, protection from external action
   D. Protect from phagocytosis
   E. Participate in metabolism
   F. Participate in breathing
51. Choose bacteria spore-forming conditions:
   A. When bacteria get into the human or animal body
   B. When factors of the external environment are unfavorable
   C. Both are correct
52. Minimal content of free water, high lipid content, high content of calcium salts, the presence of dipicolinic acid determine the stability of the ### of bacteria in the environment.
   ...
53. What group of bacteria (according their shape) can form spores?
   Rod*
54. Choose microorganisms that cannot form spores:
   A. Bacillus anthracis
   B. Clostridium perfringens
   C. Salmonella typhi
   D. Clostridium tetani
   E. Clostridium botulinum
55. Choose microorganisms that can form spores:
   A. Vibrio cholerae
   B. Clostridium perfringens
   C. Corуnebacterium diphtheria
   D. Clostridium botulinum
   E. Staphylococcus aureus
   F. B, D are correct
   G. B, C are correct
   H. All of above are correct
56. Rod-shaped spore-forming microorganisms include both ### and ###.
   ...
57. Name the Genus of spore-forming microorganisms with spore smaller than a bacterial cell in diameter. ###
   ...
58. Name the Genus of spore-forming microorganisms with spore bigger than a bacterial cell in diameter. ###
   ...
59. Name the microorganisms (in Latin) with terminal- located spore. ### ###
   ...
I.C, D are correct
60. Choose bacteria with subterminal-located (rarely central) spore bigger that a cell diameter:
   A.Klebsiella pneumoniae
   B.Corуnebacterium diphtheriae
   C.Clostridium perfringens
   D.Clostridium tetani
   E.Clostridium botulinum
   F.Bacillus anthracis
   G. C, E are correct
   H.A, B, F are correct
61. 67. Bacteria of Genus ### have central-located spore smaller than bacteria diameter that don't deform the cell.
   ...
62. Choose structural components that Spirochetes do not have:
   A. Flagella, spores
   B.Nucleoid
   C.Ribosomes
   D.Cytoplasm, plasma membrane
   E.Cell wall
   F.Endoflagella
63. Spirochetes move due to:
   A.Flagella
   B. Endoflagella
   C.Pili
   D.Fimbria
64. Name Spirochetes genus (Latin) that is characterized by numerous primary curls and the secondary curls in the forms of letters S and C. ###
   ...
65. Name Spirochetes genus (Latin) that is characterized by 3-10 large uneven curls. ###
   ...
66. Name Spirochetes genus (Latin) that is characterized by 8-12 small even curls. ###
   ...
67. Name Spirochetes genus that shows pale pink color in Romanowsky-Giemsa stain. ###
   ...
68. Name Spirochetes genus that shows blue-purple color in Romanowsky-Giemsa stain. ###
   ...
69. Human pathogenic Spirochetes belong to Treponema, Borrelia and ### genera.
   ...
70. Choose Treponema species that is pathogenic for human:
   А. Treponema pallidum
   B.Treponema macrodentium
   C.Treponema denticola
   D.Treponema orale
71. Syphilis, relapsing fever, leptospirosis are diseases caused by pathogenic ###.
   ...
72. A culture of microorganisms of the same species obtained from a single bacterial cell is called ### ###.
   ...
73. A species population isolated from different sources or from one object at different times is called ###.
   ...
74. ### ### is a set of individuals of the same species of microbes grown on nutrient medium.
   ...
75.Установите соответствие Establish a correspondence between microorganism species, shape and bacteria arrangement in a smear: 1) Neisseria meningitidis, 2) Bacillus anthracis, 3) Staphylococcus epidermidis
    The bean-shaped diplococci
    Cocci arranged in grape-like clusters
    Large rods arranged in the chain (Streptobacillus)
76.Установите соответствие Establish a correspondence between microorganism species and Gram staining:1) Vibrio cholera, 2) Treponema pallidum, 3) Corynebacterium diphtheriae
    Gram staining is not used
    Gram-negative
    Gram-positive
77.Установите соответствие Establish a correspondence between microorganism species and spore location:1)Clostridium perfringens, 2) Bacillus anthracis, 3) Clostridium tetani, 4) Mycobacterium tuberculosis
    Non-spore forming
    Sub-terminal, in the form of “spindles”
    Central
    Terminal, in the form of “drum sticks”
78.Установите соответствие Establish a correspondence between microorganism species and capsule:1) Staphylococcus aureus, 2) Bacillus anthracis, 3) Neisseria meningitides, 4) Vibrio cholerae
    No capsule
    Microcapsule
    Distinct polysaccharide capsule
    Distinct peptide containing capsule
79.Установите соответствие Establish a correspondence between microorganism species and disease: 1) Coxiella burneti, 2) Salmonella Typhi, 3) Borrelia recurrentis
    Typhoid fever
    Q-fever
    Relapsing fever
80.Установите соответствие Establish a correspondence between microorganism species and bacteria arrangement in a smear: 1) E. coli, 2) Corinebacterium diphtheriae, 3) Streptococcus pyogenes, 4) Neisseria meningitidis
    In pairs
    Chain
    At an angle forming letters X, V
    Randomly
81.Установите соответствие Establish a correspondence between microorganism species and means of motility:1) Spirochetes, 2) Vibrio, 3) Chlamydia
    Flagella
    Non-motile
    Endoflagella
82. The main tasks of medical Microbiology:
   A. Study of human pathogenic microorganisms
   B. Study of virus animal diseases pathogens and methods of their diagnostics and prevention
   C. Development of methods of prevention of infectious diseases spreading
   D. Development of methods of etiological treatment of infectious diseases
   E. Study of microorganisms involved in the synthesis of biologically active compounds, and study of biotechnologies for their production
   F. Study of microorganisms involved in matter cycling
   G. All of above are correct
83. Choose characteristics of microorganisms that are not in the basis of modern taxonomy:
   A. Morphological
   B. Serological
   C. Biochemical
   D. Physiological
   E. Molecular biological
   F. Clinical
84. Choose microbes:
   A.Prokaryotes
   B.Prions
   C.Eukaryotes
   D.Viruses
   E.All of above are correct
   F. A, C, D are correct
85. Choose cellular microorganisms:
   A. True bacteria (eubacteria)
   B. Prions
   C. Archaebacteria
   D. Eukaryotes
   E. A, C, D are correct
   F. All of above are correct
86. Domain “Eucarya” (eukaryotes) includes:
   А. Fungi
   B. Protozoa
   C. Bacteria
   D. All above are correct
   E. A, B are correct
87. Choose prokaryotic domains:
   А. Fungi
   B. Bacteria
   C. Archaea
   D. A, C are correct
   E. B, C are correct
   F. All of above are correct
88. Choose main functions of bacterial spore:
   A. Provides adhesiveness
   B. Protects from adverse environmental factors
   C. Participates in genetic material transfer process
   D. Produce enzymes
89. What method is used for capsule detection? ###-###
   ...
90. Choose organiods that prokaryotic cells do not have:
   A. Plasmid
   B. Mesosoma
   C. Mitochondria
   D. Nucleoid
91. Choose functions of intracellular inclusions of bacterial cells:
   A. Protection from phagocytosis
   B. Protection from adverse environmental factors
   C. Source of nutrients
   D. Source of oxygen
   E. Place of sporulation
92. Flagella are built from protein called ###.
   ...
93. Acid-fast bacteria are detected with ###-### method.
   ...
94. In Burri-Gins method bacterial capsules are stained:
   A. Purple
   B. Red
   C. Not painted
   D. Blue
   E. Yellow
95. In Ziehl-Neelsen method acid-fast bacteria are stained ### color.
   ...
96. The main chemical component of bacterial cell’s cytoplasmic membrane is ###.
   ...
97. Optical microscope provides a magnification of bacteria in:
   A. 1000x
   B. 10000x
   C. 100000x
98. In immersion microscopy objective lens of ### magnification are used.
   ...
99. ### is applied on microscopic slide to work with microscope immersion system.
   ...
100. What type of microscope is used to distinguish cellular structures with nanometric size?
   Electron*
101. Immersion oil is used for ### magnification.
   ...
102. The cell wall of Gram-### bacteria contains 1 layer of peptidoglycan and lipopolysaccharides.
   ...
103. Staining of bacteria by Gram is due to ### ### presence.
   ...
104. Gram-negative bacteria partially without cell wall and unable to reproduce in vitro are called ###.
   ...
105. Choose unessential component of bacterial cell:
   A. Nucleoid
   B. Capsule
   C. Cytoplasmic membrane
106. Choose characteristics of smear preparation:
   A. Drying after fixation
   B. Staining after drying
   C. Fixation after drying
   D. Fixation after staining
   E. All the answers are wrong
107. Discoloration by ### is the stage of Gram staining that allows to different bacteria into Gram-positive and Gram-negative.
   ...
108. Spherical bacteria arranged in chains are called ###.
   ...
109. Spherical bacteria arranged grape-like clusters are called ###
   ...
110. Choose spore-forming bacteria:
   A. Bacillus anthracis
   B. Typhoid bacillus
   C. Escherichia coli
   D. Vibrio cholerae
111. In Gram staining Gram-negative bacteria are stained in ### color.
   ...
112. Spherical bacteria arranged in bales or packages are called ###.
   ...
113. Existence in the form of elementary and reticular cells and reproduction only within the host cell are characteristics of ###.
   ...
114. Undulating membrane, cilia and pseudopodia are motile organelles of ###.
   ...
115. ### causative agents have the highest speed among bacteria.
   ...
116. The body of a fungus composed of interwoven threads (hyphae) is called ###.
   ...
117. Higher fungi are those haved ### mycelium.
   ...
118. Single-celled fungi round shaped are called ###.
   ...
119. Name Gram-positive soil bacteria growing in the form of thin filaments resembling the fungi mycelium; they are mostly antibiotics producers. ###
   ...
120. ### species differ from most of bacteria in cell wall absence.
   ...
121. Name bacteria these are obligate intracellular parasites causing eyes and urogenital tract lesions in humans. ###
   ...
122. The mobility of the most of bacteria is due to ### presence.
   ...
123. The purposes of ### are attaching of the smear to the glass, killing bacteria and making them more susceptible to a dye.
   ...
124. The presence of a differentiated nucleus is not typical for ###.
   ...
125. Unlike Gram-positive bacteria, Gram-negative bacteria do not have ### acids in the cell wall.
   ...
126. What is the main taxon of prokaryotes including the organisms with the most similar qualities? ###
   ...
127. Within the species microorganisms cannot differ in:
   A. Gram-stain
   B. Virulence
   C. Antigenic structure
   D. Biochemical properties
   E. Bacteriophage sensitivity
128. The bundle of shrinking fibers extending along the microbial cells in the periplasmic space and performing supporting and locomotor functions is characteristic of ### bacteria.
   ...
129. Amphitricha are bacteria with bipolar located ###.
   ...
130. Organelle that is found at the base of flagellum is called ### ###.
   ...
131. Mycosis is infectious disease caused by pathogenic ###.
   ...
132. Protein-like infectious particles without nucleic acids are called ###.
   ...
133. Sporulation is the way of ### of fungi.
   ...
+I. E. A, D, F are correct
134. Choose features of conditionally pathogenic microorganisms (compared with the pathogenic microorganisms):
   A. have low virulence to humans or show it only under certain conditions
   B. cause dangerous infections
   C. don't have a strict organ tropism, i.e. they can cause infectious inflammation in any organs and tissues
   D. are able to penetrate the macroorganism almost all possible mechanisms and pathways
   E. have a high variability in the environment
   F. often acquire antibiotic resistance, including multiple
   G. A, B, D are correct
   H. A, B, E are correct
   I. E. A, D, F are correct
135. Essential component of bacterial cell consisting of colloid phase, DNA, RNA and ribosomes is called ###.
   ...
136. What shapes of bacteria can sporulate?
   Rod*
137. Mycota is a scientific name of ###.
   ...
138. Choose characteristics of viruses:
   A. Cellular structure
   B. DNA or RNA presence
   C. DNA and RNA presence
   D. Intracellular parasitism
   E. Absence of nutrient synthesis system and accumulating energy system
   F. A, C, D are correct
   G. C, D, E are correct
   H. B, D, E are correct
139. Viruses differ from bacteria in:
   A. Size
   B. Way of reproduction
   C. Respiration absence
   D. All of above are correct
140. Choose characteristics of virion:
   A. Nucleocapsid
   B. Mitochondria
   C. Intracellular inclusions
   D. Enzymes
   E. Nucleus
   F. B, C are correct
   G. D, E are correct
   H. A, D are correct
141. Virion is:
   A. Active form of the virus
   B. Inactive form of the virus
   C. Pure viruses culture
   D. Extracellular form
   E. Accumulation of viruses
   F. A, C are correct
   G. B, D are correct
142. Choose characteristics of simple viruses:
   A. Genome
   B. Nucleus
   C. Capsid shell
   D. Matrix proteins
   E. Mitochondria
   F. All above are correct
   G. A, C are correct
143. Choose characteristics of complex viruses:
   A. DNA or RNA
   B. Nucleoid
   C. Capsid shell
   D. Supercapsid shell
   E. Cell wall
   F. B, C, E are correct
   G. A, B, D are correct
   I. A, C, D are correct
144. Viral reproduction is based on:
   A. Division
   B. Conjugation
   C. Mitosis
   D. Segmentation
   E. Disjunction
145. Choose growth media for viruses:
   A. Loewenstein-Jensen medium
   B. Developing chick embryo
   C. 199 medium
   D. Tissue culture cells
   E. Body of the sensitive laboratory animal
   F. A, C, D are correct
   G. B, C, E are correct
   H. B, D, E are correct
146. Choose the main pathogenic factor of Gram-negative bacteria:
   A. Spores
   B. Endotoxin
   C. Cytoplasm
   D. Inclusions
   E. Flagella
147. Choose the main pathogenic factor of Gram-positive bacteria:
   A. Cell form
   B. Spores
   C. Exotoxin
   D. Endotoxin
   E. Flagella
148. Choose the measure of the microorganisms pathogenicity :
   A. Virulence
   B. Specificity
   C. Commensalism
   D. Organotropic factor
   E. Parasitism
149. Choose possible sources of infection:
   A.Dirty dishes
   B.Domestic usage
   C. Bacilli carrier
   D. Milk
   E. Toys
150. Choose the term characterized by pathogens circulation in the bloodstream:
   A. Exogenous infection
   B. Autoinfection
   C. Reinfection
   D. Bacteremia
   E. Sepsis
151. Choose the term characterized by pathogens circulation and reproduction in the bloodstream:
   A. Exogenous infection
   B. Autoinfection
   C. Reinfection
   D. Bacteremia
   E. Sepsis
152. Repeated symptoms of infectious disease caused by the same pathogen without repeated infection is called:
   A. Monoinfection
   B. Secondary infection
   C. Superinfection
   D. Relapse
   E. Reinfection
153. Choose the main pathogenicity factors of Gram-negative bacteria:
   A.Spores
   B. Endotoxin
   C.Cytoplasm
   D.Inclusions
   E.Flagella
154. Choose the main pathogenicity factors of Gram-positive bacteria:
   A. Cell form
   B. Spores
   C. Exotoxin
   D. Endotoxin
   E. Flagella
155. Repeated infection caused by the same pathogen:
   A. Monoinfection
   B. Secondary infection
   C. Superinfection
   D. Relapse
   E. Reinfection
156. It is necessary for all nutritious media to have:
   A. Sources of carbon and nitrogen, water and mineral compounds
   B. Sources of hydrogen and oxygen
   C. Sources of carbon and oxygen
   D. Sources of carbon and nitrogen, amino acids, blood serum
157. Name the microbes requiring the presence of 20 % oxygen in the environment. ###
   ...
158. Name the microbes that exist only in anoxic conditions. ###
   ...
159. Choose impossible mechanism of nutrients transport into the cell:
   A. Active transport
   B. Simple diffusion
   C. Transduction
   D. Facilitated diffusion
160. Choose microorganisms that must be cultivated in anaerostat:
   A. Facultative anaerobes
   B. Strict anaerobes
   C. Strict aerobes
   D. Mesophiles
161. Choose process with bacteriophage participation:
   A. Reparation
   B. Transduction
   C. Transformation
   D. Conjugation
162. Choose treatment that do not use for sterilization:
   A. Boiling
   B. Treatment with dry heat
   C. Treatment with steam under pressure
   D. Irradiation by Gamma rays
163. Choose treatment that is used to sterilize disposable instruments in industry:
   A. Steam under pressure
   B. Fluid vapor
   C. Dry heat
   D. Gamma radiation
164. Choose bacteria that are not sanitary significant microbes of water:
   A. General coliform bacteria
   B. Thermotolerant coliform bacteria
   C. Coli-phages
   D. Hemolytic Streptococci
165. Choose antibiotics violating the synthesis of peptidoglycan:
   A. Ampicillin, oxacillin, penicillin
   B. Tetracycline, oleandomycin, cephalosporins
   C. Erythromycin, chloramphenicol, polymixin
166. Choose fluoroquinolones action mechanism:
   A. Violation of protein synthesis on the ribosomes of bacterial cells
   B. Violation of protein synthesis at the transcriptional level
   C. Violation of the structure and function of bacterial DNA
   D. Violation of the permeability of bacterial cytoplasmic membrane
167. The first antibiotic – penicillin –was discovered by:
   A. Ehrlich
   B. Paster
   C. Ivanovsky
   D. Fleming
168. Choose organelles of prokaryotic cell:
   A. Nucleus
   B. Golgi Apparatus
   C. Nuclear membrane
   D. Mesosome
   E. Mitochondria
169. The morphological study of bacteria requires:
   A. Electron microscope
   B. Unstained smears
   C. Immersion microscopy method
   D. Low magnification microscope
170. Choose essential components of bacterial cell:
   A. Flagella
   B. Nucleoid
   C. Spore
   D. Cytoplasmic membrane
   E. Cytoplasm
   F. A, B, C are correct
   G. C, D, E are correct
   H. B, D, E are correct
I. A, C, E are correct
171. Choose main morphological groups of bacteria:
   A. Spiral
   B. Cocci
   C. Rods
   D. Vibrio
   E. Curved rods
   F. Filamentous shape
   G. A, B, C are correct
   H. D, E, F are correct
172. Choose basis of medical important cocci classification:
   A. Size
   B. Number and location of flagella
   C. Division in different planes
   D. Differences in capsule formation
   E. Gram staining.
   F. B, D are correct
   G. A, B are correct
   H. C, E are correct
173. Choose characteristics of Streptococci:
   A. Sporulation
   B. Division in one plane
   C. Presence of flagella
   D. Grapes-like form
   E. Chain-like form
   F. A, C are correct
   G. B, E are correct
   H. C, D are correct
174. Choose characteristics of Bacillus species:
   A. Inclusion of volutine grains
   B. Cylindrical shape
   C. Gram-negative stain
   D. Spores
   E. Gram-positive stain
   F. A, B, C are correct
   G. B, D, E are correct
175. Choose characteristics of smear preparation:
   A. Use of pre-heat killed bacteria
   B. Fixation by the flame
   C. Drying the smear in air
   D. Drying the smear in the flame
   E. Fixation by the drying in air
   F. A, B are correct
   G. B, C are correct
   H. D, E are correct
176. Choose characteristics of bacterial cell:
   A. Differentiated nucleus
   B. Diffuse-located nuclear substance
   C. Cell wall absence
   D. Cytoplasm is surrounded by a multilayered shell
   E. Presence of spare nutrients in the cytoplasm
   F. A, C, E are correct
   G. B, D, E are correct
177. Choose inclusions of microbial cell:
   A. Vacuoles
   B. Lipid drops
   C. Volutin granules
   D. Ribosomes
   E. Glycogen and starch granules
   F. All above are correct
   G. B, C, E are correct
178. Choose characteristics of cytoplasmic membrane:
   A. Participates in protein synthesis
   B. Shape bacteria
   C. Osmotic barrier of the cell
   D. Regulates cell metabolism
   E. Protects bacteria from negative external influence
   F. B, D, E are correct
   G. A, C, D are correct
179. Mycobacteria are acid-fast bacteria due to:
   A. Cytoplasmic membrane specificity
   B. Capsule presence
   C. Carbohydrates
   D. Сhitin-like substances
   E. Lipids and waxes presence
180. Choose stain method that is used to detect acid-fast bacteria:
   A. Gram stain
   B. Romanowsky-Giemsa stain
   C. Ziehl-Neelsen stain
   D. Loeffler stain
   E. Neisser stain
181. Choose bacilli spore significance:
   A. Reproduction
   B. Accumulation of reserve nutrients
   C. Conservation of the species in adverse conditions
   D. Protective response when injected into the host
   E. Sign of cell degeneration
182. Choose sporulation conditions:
   A. Unfavorable external environment
   B. Encounter into the human or animal body
   C. Lack of the nutrients
   D. Prolonged cultivation on nutrient media without reseeding
   E. Drying
   F. Mode of reproduction
   G. B, D, F are correct
   H. A, C, D, E are correct
183. Choose types of bacteria that involve spore-forming species:
   A. Cocci
   B. Rickettsia
   C. Bacillus
   D. Viruses
   E. Clostridium
   F. A, B are correct
   G. C, E are correct
184. Choose diseases that are caused by spore-forming causative agents:
   A. Diphtheria
   B. Typhoid
   C. Tetanus
   D. Anthrax
   E. Botulism
   F. A, B, C are correct
   G. C, D, E are correct
185. Such qualities as absence of growth and binary fission possibility, one type of nucleic acid, absolute intracellular parasitism are characteristics of Kingdom ###
   ...
186. Virion of 42 nm in size is characteristic of ### ### virus.
   ...
187. What does not present into simple virus components?###
   ...
188. Formation of Filatov-Koplik spots inside the mouth mucosa is characteristic of ### (disease).
   ...
189. The structure of a complex virion includes all, except:
   A. Nucleic acid
   B. Capsule
   C. Capsid consisting of capsomeres
   D. Supercapsid
190. The type of virion’s capsid symmetry when capsomeres follow the curves of nucleic acid is called ###.
   ...
191. The type of virion’s capsid symmetry when capsomeres surround nucleic acid in polyhedron shape is called ###.
   ...
192. Positive RNA genome (RNA+) of viruses act during infection as:
   A. mRNA (transmits information to the ribosomes)
   B. Antigen
   C. All above are correct
193. Reverse transcriptase, RNA-asa and DNA- polymerase are enzymes of the ### Family.
   ...
194. Установите верную последовательность The sequence of interaction of the virus with the cell and the reproductive process is 1). Adsorption; 2). Replication of viral genome and Synthesis of viral Proteins; 3). Release of viral particles from the cells; 4). Virion assembly; 5). Penetration of the virus into the cell; 6). Deproteinization of virion.
   1, 5, 6, 2, 4, 3
195. Membranes fusion or endocytosis are the ways of penetration of the ### into host cells.
   ...
196. Name the cells’ protein that is formed under the influence of an interferonogene (virus, etc.) and protect cells from viral infection.
   Interferon
197. Interferon protects cells from the viral infection by:
   A. Neutralization of the virus
   B. Indirectly interrupting information from the virus genome to the ribosomes
   C. Activate antibodies neutralizing action
198. Choose method that do not use for laboratory diagnostics of viral infections:
   A. Microscopical (detection of intracellular inclusions, RIF)
   B. Virological method (cultivation of viruses in chicken embryo, in cell culture, in laboratory animals)
   C. Allergological method
   D. Serological method
   E. Detection of viral antigens using highly sensitive tests (ELISA, RIA)
   F. Nucleic acid probes, PCR
199. Установите верную последовательность The sequence of the stages of interaction of phages with bacterial cell: 1). Adsorption; 2). replication of nucleic acid; 3). release of Mature particles; 4) syntesis of phages proteins; 5) penetration of phages genom
   1, 5, 2, 4, 3.
200. ### penetrates into bacterial cell by injecting the nucleic acid through the appendage channel.
   ...
201. The interaction of bacteriophage with bacterial cell, resulting in bacteriophage DNA embedding in the bacterial genome is called ### type of infection.
   ...
202. A productive type of infection, resulting in formation of phage progeny and lysis of the bacteria is characteristic of ### bacteriophage.
   ...
203. Integrative type of infection with the formation of a prophage is characteristic of ### bacteriophage.
   ...
204. Choose impossible way of bacteriophages practical application:
   A. Bacterial cultures phagotyping
   B. Indication of bacteria in the environment
   C. Creation of artificial active immunity
   D. Application for therapy and prevention of diseases
205. Choose disease that is not treated with bacteriophage medication:
   A. Influenza
   B. Dysentery
   C. Salmonellosis
   D. Purulent infection
206. ### is used with epidemiological purpose to establish the source of infection using bacteriophages.
   ...
207. Influenza virus ultrastructure includes:
   A. Fragmented RNA
   B. Nonfragmented RNA
   C. Double-stranded RNA
   D. DNA
   E. Capsid
   F. Supercapsid
   G. B, C, D are correct
   H. A, E, F are correct
208. Choose the major antigens of influenza virus types A and B:
   A. Hexon-antigen
   B. Hemagglutinin (HA)
   C. Neuraminidase (NA)
   D. Nucleoprotein (NP)
   E. M-antigen (matrix protein associated with the NP)
   F. Fusion and hemolysis proteins
   G. B, C, D, E are correct
   H. A, B, C are correct
209. Hemagglutinin and neuraminidase of influenza virus are ### located antigens.
   ...
210. Nucleoprotein (NP) and M-matrix protein of influenza virus are ### located antigens.
   ...
211. Choose properties of hemagglutinin (H-antigen) of influenza virus:
   A. Sticks red blood cells together, is detected by RHA
   B. Is detected by its reaction with sialic acids
   C. Conservative
   D. Induces non-protective antibodies formation
   E. Undergoes antigenic shift and antigenic drift
   F. Participates in the process of virus adsorption onto the cell
   G. Basis of influenza virus type A classification
   H. B, C, D, F are correct
   I. A, E, F, G are correct
212. What types of influenza virus do you know? ###, ###, ###
   ...
213. Influenza A virus is divided into subtypes, except:
   A. А (H1N1)
   B. А (H3N3)
   C. А (H2N2)
   D. А (H3N2)
214. What test is used to determine of types of influenza virus?
   A. Complement fixation reaction (CFR)
   B. Neutralization reaction
   C. Hemagglutination reaction
   D. Reaction of indirect hemagglutination
215. Choose features of immune response to influenza:
   A. Does not form
   B. Type-specific
   C. Is formed in presence of antibodies to hemagglutinin and neuraminidase
   D. Is formed in presence of antibodies to ribonucleoproteins
   E. Depends significantly on the presence of secretory immunoglobulins A
   F. Intense, long-lasting
   G. B, C, E are correct
   H. A, D, F are correct
216. Pandemics and epidemics are mostly caused by influenza virus type ###.
   ...
217. Epidemics and local outbreaks are mostly caused by influenza virus type ###.
   ...
218. Sporadic diseases are caused by influenza virus type ###.
   ...
219. What biomaterials are used for viroscopical and virological diagnostics of influenza? ### ###
   ...
220. Choose biomaterials that are used for serological diagnostics of influenza?
   A. Two blood samples (double serum)
   B. A single blood sample (serum)
   C. Nasopharyngeal swabs
   D. Feces
221. Choose medication that is used for active specific prevention of influenza:
   A. Live intranasal vaccine
   B. Inactivated virion vaccine
   C. Subunit vaccine
   D. Sabin vaccine
   E. Salk vaccine
   F. Live peroral vaccine for children
   G. A, B, C are correct
   H. D, E, F are correct
222. Measles virus contains a negative-sense spiral single-stranded ###.
   ...
223. Antigens of measles virus according to variability are:
   A. Stable ones
   B.Variable ones
224. What is not the characteristic of the measles virus:
   A. A, B, C serotypes
   B. Serotypes are not detected
   C. Antigens unity of viruses from different geographical areas is observed
225. Cytopathogenic effect of measles virus cultivated in cell cultures is ### formation
   ...
226. Slow infections, multiple sclerosis and subacute sclerosing panencephalitis (SSPE) can develop several years after ### infection
   ...
227. Choose medication that is used for active prevention of measles in Russia
   A. Inactivated vaccine
   B. Live vaccine
   C. Hemagglutinin-split vaccine
228. Polioviruses , Coxsackieviruses and ECHO viruses belong to the genus ### in the family ###.
   ...
229. Choose characteristics of polioviruses:
   A. Contain positive-sense RNA genome
   B. Contain negative-sense RNA genome
   C. Have capsid with icosahedral symmetry
   D. Have supercapsid
   E. Viral particle size is 30 nm
   F. Viral particle size is 300 nm
   G. Bullet shape
   H. B, D, F, H are correct
   I. A, C, E, G are correct
230. What are the types of poliovirus?###, ###, ###
   ...
231. Choose the impossible route of poliomyelitis transmission:
   A. Fecal-oral
   B. Vector-borne
   C. Droplet
   D. Food-borne (water, milk and butter consumption)
232. Poliovirus is most likely to be isolated from ###.
   ...
233. Select characteristics of immunity in case of poliomyelitis:
   A. Type-specific
   B. Humoral
   C. Cell-mediated
   D. Forms with secretory immunoglobulins A significant participation
   E. Forms with DTH T-effectors significant participation
   F. C, E are correct
   G. A, B, C are correct
234. Live vaccine for poliomyelitis prevention is given ### ### (Latin term).
   ...
235. Choose vaccine for active specific prevention of enterovirus infections that is not developed:
   A. Inactivated poliovirus vaccine
   B. ECHO virus vaccine
   C. Live poliovirus vaccine
236. Choose virus that is not causative agent of viral hepatitis:
   A. Hepatitis virus except hepatitis A and B
   B. Hepatitis A virus
   C. Epstein-Barr virus
   D. Hepatitis B virus
   E. Hepatitis D virus
   F. Hepatitis C virus
   G. Hepatitis E virus
   H. B, D are correct
237. What is the reservoir of hepatitis A virus? ### ###
   ...
238. Choose methods that are used for hepatitis A diagnostics:
   A. Virusoscopy (IEM)
   B. Virusological method, cell culture contamination
   C. Serological method, specific IgM determination
   D. HBs antigens determination
   E. Virus determination by ELIZA, RIA
   F. A, C, E are correct
   G. B, C, D are correct
239. What type of hepatitis virus belongs to the family of Hepadnaviruses, contains DNA, is transmitted parenterally, is not cultivated in cell culture, is oncogenic? ### (letter)
   ...
240. HBs antigen of hepatitis B virus is ### located antigen
   ...
241. НBе Ag of hepatitis B virus is a fragment of ### antigen.
   ...
+ I. A, D, E, F, G are correct
242. Choose characteristics of hepatitis B virus:
   A. Is not inactivated by treatment at 60 C for several hours
   B. Is inactivated by treatment at 60 C for several hours
   C. Is inactivated by treatment at 100 C for 15-20 minutes
   D. Is not inactivated by treatment at 60 C for 15-20 minutes
   E. UV resistant
   F. Formalin sensitive
   G. Detergents sensitive
   H. B, C, E are correct
   I. A, D, E, F, G are correct
243. Select characteristics of immunity in case of hepatitis B:
   A. Humoral
   B. Cell-mediated
   C. The role of antibodies to HBsAg is significant
   D. Antibodies to HBsAg are not protective
   E. Protects from repeated infection
   F. Does not protect from repeated infection
   G. B, D, F are correct
   H. A, C, E are correct
244. Hepatitis D virus superinfection occurs in the presence of hepatitis ### virus (letter)
   ...
245. The rabies virus belongs to ### genus in the ### family.
   ...
246. The negative-sense single-stranded no fragmented RNA, nucleocapsid with spiral symmetry, outer-shell, capsid proteins and matrix proteins, RNA-polymerase, glycoprotein and a bullet shape are characteristics of ### virus.
   ...
247. What is not the reservoir of Rabies virus in nature:
   A. Rodents
   B. Infected dogs, foxes, wolves, jackals, cats, bobcats
   C. Bats
248. Rabies virus can be transmitted throught ### and ### of animal.
   ...
249. Rabies incubation period when the virus penetrates through head and face skin is:
   A. 10-14 days
   B. More than 1,5 month
250. Special cylinders virus promotion from sensitive peripheral nerves endings to perineural spaces and brain and spinal cord neurons and virus reproduction in neurons with formation of Babesh-Negrie cytoplasmic inclusions play significant role in pathogenesis of ### infection.
   ...
251. “Fixed” rabies virus invented by Pasteur for vaccination does not differ from rabies virus in:
   A. Plaque formation in the chorion-allantois shell of infected chicken embryos
   B. Peproduction only in brains of rabbits
   C. Non-pathogenicity for people and dogs
   D. Causing infection with stable incubation period when passaging in rabbit’s brains
252. Choose indications for anti-rabies immunoglobulin treatment:
   A. Multiple bites, dangerously-located (head, neck)
   B. Anamnesis data about past rabies infections
253. HIV genome includes two single-strand ### molecules. (abbreviation)
   ...
254. HIV virion supercapsid membrane contains glycoproteins gp with molecular masses of ### and ###.
   ...
255. Gp 120 of HIV virion attaches to the host cell membrane in the presence of ### receptor on it.
   ...
256. HIV penetration into the cell cannot be achieved by:
   A. Virogeny
   B. Membranes fusion
   C. Receptor endocytosis
257. HIV infection specifically affects ### system of the organism.
   ...
258. What is impossible route of HIV transmission? ###
   ...
259. Choose cells that are not HIV target-cells:
   A.T-helper cells
   B. Monocytes, macrophages
   C. Hepatocytes
   D. Langerhans cells
   E. Astrocytes, endothelial and epithelial cells
260. Choose biomaterials that contains insufficient amount of HIV:
   A.Blood
   B. Semen
   C. Vaginal and cervical discharge
   D. Brest milk
   E. Saliva, urine, lacrimal fluid
261. The goal of HIV infection therapy is:
   A. Prevention of disease progression
   B. Complete curing
   C. The elimination of the virus from the body
262. Name the form of a virus that exists outside the host cell:
   Virion
263. Phased papular rash is the characteristic of ### virus infection.
   ...
264. Measles mild form that vaccinated people may have is called ### measles.
   ...
265. ### type of virus-eukaryotic cell interaction can be characterized by viral DNA integration into the cell chromosome.
   ...
266. What is the major marker of human HBV infection?
   HBS Ag
267. What causative agents of infectious diseases do not have cellular structure?
   Virus
268. What viruses contain virions with the enzyme “reverse transcriptase”?
   Retroviruses
269. HIV attach to the ### subpopulations of T-lymphocytes.
   ...
270. Choose the reason of liver cells cytolysis in case of viral hepatitis B:
   A.The impact of the virus on hepatocytes
   B. Immune response to viral antigens and autoantigens
   C. Bile ducts lesions
271. Chronization of pathological process causing cirrhosis is mostly characteristic of viral hepatitis ###.
   ...
272. Choose characteristic of influenza pathogens antigenic drift:
   A. Antigenic variations of the viral hemagglutinins are insignificant
   B. Is associated with insignificant antigenic variations of viral capsid proteins
   C. The main reason of pandemics
   D. The result of recombination of human and bird influenza viruses
   E. Causes new antigenic types of influenza viruses formations
273. The productive interaction of the simple virus with the host cell results in ### of the cell.
   ...
274. Influenza viruses are mostly cultivated on ### ###.
   ...
275. What reaction is used to determine of influenza virus in infected chicken embryos? ###
   ...
276. Damage of medulla oblongata of the motor neurons and the front horns of spinal cord is typical for pathogenesis of ###.
   ...
277. Poliomyelitis vaccine injection provides prevention of ### clinical form of infection.
   ...
278. What type of nucleiс acid is typical for hepatitis B virus? ### (abbreviation)
   ...
279. Hepatitis C virus belongs to ### family.
   ...
280. What type of nucleiс acid presents in hepatitis C virus? ### (abbreviation)
   ...
281. Hepatitis C virus can hide from immune system due to its high ###.
   ...
282. Multiyear latent progress is characteristic of hepatitis ### virus.
   ...
283. What type of hepatitis virus requires assistance of helper-virus? ###
   ...
284. What is the reservoir of parental virus hepatitis? ###
   ...
285. Gp120 and gp41 are antigens of ### (abbreviation) virus.
   ...
286. What HIV receptor provides interaction with target-cells?
   gp120
287. Indication of HIV infection is required ### determination in patient’s blood.
   ...
288. What type of nucleiс acid presents in influenza virus? ### (abbreviation)
   ...
290. Hepatitis B virus testing of donated blood is carrying out through ### (abbreviation) antigen determination.
   ...
291. What is the major mechanism of hepatitis A virus transmission? ###-###
   ...
292. Bacteriophages wich lyse only definite bacteria within the species are called ### bacteriophages
   ...
293. Choose enzymes of HIV pol-complex:
   A. Protease, integrase, RNA-ase
   B. DNA-polymerase, integrase
   C. DNA-polymerase, protease
   D. Reverse transcriptase, RNA-ase, DNA-polymerase, integrase
294. Choose the measles symptoms:
   A. Jaundice, liver and spleenincrease
   B. Phased skin rash
   C. Parotid salivary glands swelling
   D. Paralysis development
295. What is not typical for hepatitis D virus:
   A. Defective DNA containing virus
   B. Reproduction is possible only during presence of hepatitis B virus
   C. The virus monoinfection is possible
   D. The disease develops as co-infection or superinfection in presence of hepatitis B virus
296. Choose the virus that can be isolated from the feces 3 weeks after infection:
   A. Hepatitis B virus
   B. Poliovirus
   C. Influenza virus
   D. Measles virus
297. What enzyme presents in the core of the influenza virus?
   A. Reverse transcriptase
   B. DNA-polymerase
   C. RNA- polymerase
   D. Protease
298. Hydrophobia and aerophobia, hallucinations, alternation of excitement and depression are the features of ### disease.
   ...
299. What virus causes swelling of parotid salivary glands?
   A. Measles virus
   B. Rubella virus
   C. Influenza virus
   D. Mumps virus
300. Mitochondria are missing in
   A. Filamentous fungi
   B. Protozoan parasites
   C. Viruses
   D. Yeasts
   E. Cestodes
301. A culture isolate from a patient with subacute endocarditis is reported to be gram positive and possess a complex carbohydrate cell wall. What is the most likely taxonomic group of the causal agent?
   A. Fungus
   B. Parasite
   C. Prion
   D. Prokaryot
   E. Virus
302. A 4-year-old boy develops several honey-crusted lesions behind his ears and on his face. The simplest test for the physician to determine the genus of bacteria responsible for this child’s illness is the
   A. catalase test
   B. coagulase test
   C. growth of the organism in 6.5% sodium chloride
   D. hemolysis pattern on blood agar
   E. polymerase chain reaction
303. Sixteen residents in a retirement home have fever, malaise, and anorexia. These residents have taken their meals prepared by the same kitchen. Blood cultures from 11 of these residents grow Salmonella enterica subsp. Typhi. The primary reservoir of this organism is
   A. hen’s egg
   B. dogs and cats
   C. turkeys
   D. people
   E. water
304. What is the structure that is found in gram-negative but not in gram-positive bacteria?
   A. Capsule
   B. Cell wall
   C. Cytoplasmic membrane
   D. Endospore
   E. Outer membrane
305. The clinical laboratory reports the presence of 0157:H7 strains of E. coli in the bloody stools of 6 children ages 3–5 who attended a local petting zoo. These young children would be at an increased risk for developing
   A. buboes
   B. hemolytic uremic syndrome
   C. infant botulism
   D. renal stones
   E. rice water stools
306. A 65-year-old man develops pneumonia. The organisms isolated from the sputum are gram-positive cocci that are alpha hemolytic on blood agar and sensitive to optochin. Which structure of the causal agent provides protection against phagocytosis?
   A. Capsule
   B. Catalase
   C. Coagulase
   D. M protein
   E. Teichoic acid
307. A 68-year-old woman on chemotherapy for leukemia has developed sepsis due to an infection with Escherichia coli. The following day the patient develops septic shock and dies. The structure on the bacterium most likely responsible for causing septic shock in this patient is
   A. capsule
   B. lipopolysaccharide
   C. pili
   D. spore
   E. teichoic acid
308. What is the typical means of transmission of a toxin that blocks the release of inhibitory transmitters GABA and glycine?
   A. Eating home-canned foods
   B. Fecal-oral, travel to foreign country
   C. Infant given honey during the first year of life
   D. Puncture wound
   E. Respiratory, with incomplete vaccination history
309. A 10-year-old girl with an incomplete vaccination history presents to her pediatrician with a fever of 101.5°F (38oC), sore throat, malaise, and difficulty breathing. Physical examination reveals cervical lymphadenopathy and a gray, leathery exudate in the rear of the oropharynx. The area bleeds profusely when disturbed with a tongue depressor. Which of the following correctly describes the causal agent?
   A. Gram-negative rod; toxin that inhibits protein synthesis
   B. Gram-negative rod; toxin that increases cAMP
   C. Gram-positive aerobic rod; toxin that inhibits protein synthesis
   D. Gram-positive anaerobic rod; toxin that inhibits protein synthesis
   E. Gram-positive aerobic rod; toxin that increases cAMP
310. A 38-year-old man who recently visited India on business presents to the emergency department with profuse watery diarrhea flecked with mucus, and severe dehydration. Which of the following correctly describes the causal agent?
   A. Gram-negative curved rod; toxin that increases cAMP
   B. Gram-negative curved rod; toxin that inhibits protein synthesis
   C. Gram-negative rod; toxin that increases cAMP
   D. Gram-negative rod; toxin that inhibits protein synthesis
   E. Intoxication with a heat labile toxin that blocks the release of acetylcholine
311. Two days after eating a meal that included home-canned green beans, 3 people developed various degrees of visual problems, including double vision and difficulties focusing. Describe the Gram reaction of the organism most likely to be isolated from the leftover beans and lab findings which would be used in its identification.
   A. A gram-positive coccus which is catalase-positive and grows in a high salt environment
   B. A gram-positive aerobic bacillus which sporulates
   C. A gram-positive coccus which is catalase-negative and optochin-resistant
   D. A gram-positive bacillus grown on a low oxidation-reduction medium
   E. A gram-negative bacillus capable of reducing nitrates to nitrites
312. A 7-day-old infant presents to the emergency department with a fever, poorfeeding, and a bulging fontanelle. During her physical examination, she begins to convulse. A Gram stain of the CSF reveals gram-negative diplococci. Which of the following organisms is the most likely causal agent?
   A. Escherichia coli
   B. Haemophilus influenzae
   C. Listeria monocytogenes
   D. Neisseria meningitidis
   E. Streptococcus agalactiae
313. A 40-year-old homeless man presents to the emergency department with fever and night sweats, coughing up blood. Acid-fast bacilli are identified in his sputum. Which of the following virulence factors allows the causal agent to inhibit phagosome-lysosome fusion to survive intracellularly?
   A. Cord factor
   B. Calcium dipicolinate
   C. Peptidoglycan
   D. Sulfatides
   E. Tuberculin
314. A 45-year-old woman presents to the emergency department with intense pain in her lower back and a burning sensation upon urination. A urine culture was taken and plated on MacConkey agar. Gram-negative rods that did not ferment lactose were identified. Which virulence factor of the causal agent is most important to pathogenesis?
   A. Capsule
   B. Catalase
   C. Coagulase
   D. Exotoxin
   E. Urease
315. A 15-year-old girl develops a sore throat, fever, and earache of approximately 1 week duration. Upon examination by her physician, an erythematous rash is noted covering most of her body and her tongue appears bright red. Which of the following is the description of the causal agent?
   A. Gram-positive coccus, alpha hemolytic, catalase negative
   B. Gram-positive coccus, beta hemolytic, catalase negative
   C. Gram-positive coccus, alpha hemolytic, catalase positive
   D. Gram-positive coccus, beta hemolytic, catalase positive
   E. Gram-positive coccus, gamma hemolytic, catalase negative
316. How is a prophage created?
   A. Through activation of the recA gene product of an exogenote
   B. Through infection of a bacterial cell with a virulent bacteriophage
   C. Through site-specific recombination of a temperate phage and bacterial DNA
   D. Through excision of bacterial DNA and active lytic replication of a bacteriophages
317. The ability of a cell to bind DNA to its surface and import it is required for which genetic process?
   A. Conjugation
   B. Generalized transduction
   C. Homologous recombination
   D. Site-specific recombination
   E. Specialized transduction
   F. Transformation
318. The process by which bacterial or plasmid DNA may be mistakenly incorporated (during assembly) into one phage being produced by the lytic life cycle and then that DNA-transferred to another bacterial cell which may acquire some new genetic traits is called
   A. Conjugation
   B. Generalized transduction
   C. Homologous recombination
   D. Site-specific recombination
   E. Specialized transduction
   F. Transformation
319. Lysogenic conversion
   A. is a change in pathogenicity due to the presence of a prophage.
   B. is the induction of a prophage to its virulent state.
   C. is the conversion of a virulent phage into a temperate phage.
   D. refers to the incorporation of a prophage into the chromosome.
   E. is the immunity that a prophage confers on a bacterium.
320. Which of the following events is most likely due to bacterial transformation?
   A. A formerly non-toxigenic strain of Corynebacterium diphtheriae becomes toxigenic.
   B. A non-encapsulated strain of Streptococcus pneumoniae acquires a gene for capsule formation from the extract of an encapsulated strain.
   C. A strain of Neisseria gonorrhoeae starts producing a plasmid-encoded ?-lactamase similar to that another Gram-negative strain.
   D. A gene for gentamicin resistance from an Escherichia coli chromosome appears in the genome of a bacteriophage that has infected it.
321. Which of the following mechanisms is most likely to be involved in multiple drug resistance transfer from one cell to another?
   A. Specialized transduction of a chromosomal gene for drug resistance
   B. Transformation of chromosomal genes
   C. Transposition
   D. Conjugation with a cell with a free plasmid carrying drug resistance
   E. Conjugation with a cell with chromosomal drug resistance
322. Which of the following agents, if introduced into a growing culture of bacteria, would halt growth but, if then removed, would allow growth to resume?
   A. Antiseptic
   B. Bacteriocide
   C. Bacteriostat
   D. Disinfectant
   E. Sterilizing Agent
323. A 54-year-old man develops a pyogenic infection along the suture line after knee surgery. The laboratory gives a preliminary report of a hemolytic, catalase-positive, coagulase-positive, Grampositive coccus. The most likely causative agent is
   A. Moraxella catarrhalis
   B. Staphylococcus aureus
   C. Staphylococcus epidermidis
   D. Streptococcus agalactiae
   E. Streptococcus pyogenes
324. The reagent used to distinguish staphylococci from streptococci is
   A. Hydrogen peroxide
   B. Fibronectin
   C. Fibrinogen
   D. Oxidase
325. Negri bodies are associated with
   A. Cytomegalovirus infections
   B. Herpes simplex virus infections
   C. Rabies virus infections
   D. Rubella virus infections
326. Lymphotropic and macrophage trophic designation is important in the pathogenesis of
   A. Cytomegalovirus
   B. Herpes simplex virus
   C. Human immunodeficiency virus
   D. JC virus
327. Dane particles are associated with
   A. Hepatitis A virus
   B. Hepatitis B virus
   C. Hepatitis C virus
   D. Hepatitis E virus
328. The exchange of homologous segments of RNA between two different influenza type A viruses is called
   A. Complementation
   B. Genetic reassortment
   C. Phenotypic masking
   D. Phenotypic mixing
329. Viruses whose genomes have a messenger (positive-sense) polarity are
   A. Adenoviruses
   B. Papovaviruses
   C. Paramyxoviruses
   D. Polioviruses
330. A commercial vaccine consisting of virion subunits prepared by recombinant technology exists for
   A. Hepatitis B virus
   B. Rabies virus
   C. Rotavirus
   D. Varicella-zoster virus